K
Khách

Hãy nhập câu hỏi của bạn vào đây, nếu là tài khoản VIP, bạn sẽ được ưu tiên trả lời.

13 tháng 12 2015

\(\sqrt{1.2015}\le\frac{2016}{2}\Rightarrow\frac{1}{\sqrt{1.2015}}\ge\frac{2}{2016}\)

=>S\(\ge\frac{2.1015}{2016}\)\(>\frac{2.2014}{2015}\)

13 tháng 12 2015

Bài này dễ dùng Cô si cho mẫu là OK

5 tháng 7 2016

với \(a>0,b>0\)ta có \(\sqrt{a}.\sqrt{b}\le\frac{a+b}{2}\Rightarrow\frac{1}{\sqrt{a}.\sqrt{b}}\ge\frac{2}{a+b}\)
từ đó ta có : \(\frac{1}{\sqrt{k\left(2016-k\right)}}\ge\frac{2}{k+2016-k}\ge\frac{2}{2016}=\frac{1}{1008},\)với mọi \(k\in N^{\cdot}\)
Suy ra \(S_k\)\(\ge k.\frac{1}{1008}>k.\frac{1}{1018}\)(đpcm).

5 tháng 7 2016

ho qua

21 tháng 8 2018

Nhân cả 2 với (\(\sqrt{2015^2-1}\)+\(\sqrt{2014^2-1}\))

A = 2015^2 -1 -2014^2 + 1 = (2014 + 1)^2 -2014^2 = 2.2014 + 1

B = 2.2014

=> A = B + 1

19 tháng 7 2018

Theo BĐT \(AM-GM\) ta có : \(\sqrt{ab}< \frac{a+b}{2}\) với \(a;b>0;a\ne b\)\(\Rightarrow\frac{1}{\sqrt{ab}}>\frac{2}{a+b}\)

Áp dụng ta được : 

\(S>\frac{2}{1+2014}+\frac{2}{2+2013}+...+\frac{2}{k+2014-k+1}+...+\frac{2}{2014+1}\)

\(=2\left(\frac{1}{2015}+\frac{1}{2015}+...+\frac{1}{2015}\right)=2.\frac{2014}{2015}\)

Vậy \(S>2.\frac{2014}{2015}\)

18 tháng 7 2016

\(tacó:...\frac{1}{3.\left(\sqrt{1}+\sqrt{2}\right)}>\frac{1}{3.2}=\frac{1}{\left(1+2.1\right).2.1}\) 

\(\frac{1}{5.\left(\sqrt{2}+\sqrt{3}\right)}>\frac{1}{5.4}=\frac{1}{\left(1+2.2\right).2.2}\) 

\(\frac{1}{7.\left(\sqrt{3}+\sqrt{4}\right)}>\frac{1}{7.6}=\frac{1}{\left(1+2..3\right).2.3}\) 

....

\(\frac{1}{49.\left(\sqrt{48}+\sqrt{49}\right)}>\frac{1}{49.48}=\frac{1}{\left(1+2.48\right).2.48}\) 

cộng vế theo vế ta đươc S =\(\frac{1}{\left(1+2.1\right).2}+\frac{1}{\left(1+2.2\right).2.2}+...+\frac{1}{\left(1+2.48\right).48.2}\)

\(=\frac{1}{2}.\left(\frac{1}{3}+\frac{1}{10}+\frac{1}{21}+\frac{1}{36}+...+\frac{1}{4656}\right)\)  <  \(\frac{1}{2}.\left(\frac{1}{3}+\frac{1}{6}+\frac{1}{10}+\frac{1}{15}+...+\frac{1}{4656}\right)\)

mà lại có : \(A=\frac{1}{3}+\frac{1}{6}+\frac{1}{10}+\frac{1}{15}+..+\frac{1}{4656}\) 

=> \(\frac{1}{2}A=\frac{1}{6}+\frac{1}{12}+\frac{1}{20}+\frac{1}{30}+...+\frac{1}{9312}=\frac{1}{2.3}+\frac{1}{3.4}+\frac{1}{4.5}+...+\frac{1}{96.97}\) 

             = \(\frac{1}{2}-\frac{1}{3}+\frac{1}{3}-\frac{1}{4}+\frac{1}{4}-\frac{1}{5}+...-\frac{1}{97}=\frac{1}{2}-\frac{1}{97}=\frac{95}{194}\)  

vậy S < \(\frac{95}{194}\) 

mà \(\frac{95}{194}< \frac{3}{7}\) 

=> S < \(\frac{3}{7}\)

KẾT LUẬN  : S <\(\frac{3}{7}\)